SIE Question To Do Again

Réussis tes devoirs et examens dès maintenant avec Quizwiz!

An investor purchases 5 6-month XZY 45 calls for 3.25. Excluding commissions, how much does the investor pay for these calls? A. $1,625 B. $16.25 C. $241.35 D. $325

A. $1,625 The premium of an option contract is based on a single share. A standard contract covers 100 shares. Because this investor purchases 5 contracts, the price is 5*3.25*100 = 1625

If three people own a brokerage account that is registered as Joint Tenancy with Rights of Survivorship (JTWROS), and two of them then die at the same time, how much of the account does the survivor inherit? A. 100% B. None, because the decedents' heirs assume ownership C. 33% D. 66%

A. 100% Each owner in a JTWROS has an undivided interest in the entire property. If one dies, any remaining holder retain their rights to the entire account

An annuity owner, age 65, has chosen to annuitize a qualified annuity. Each monthly income payment will be A. 100% taxable as ordinary income B. Partially taxed as capital gains C. Partially taxed as ordinary income D. Received tax-free

A. 100% taxable as ordinary income Qualified annuities are funded with pre-tax dollars. Since neither the purchase payments nor the earnings in these annuities have been taxed, monthly payments from qualified annuities are fully taxable

A mutual fund that wishes to charge the maximum sales charge of 8.5% must offer all of the following features EXCEPT A. A level load share class option B. Automatic reinvestment of dividend and capital gains distributions at NAV C. Breakpoint schedule D. Rights of accumulation

A. A level load share class option level load share class option is not one of the features required for a mutual fund to charge the maximum sales charge. The three required features are breakpoints, reinvestment of dividends at nav, and rights of accumulation

Unlike other investment companies, a unit investment trust does not A. Actively manage its portfolio B. Provide a prospectus to investors C. Have an investment objective D. Offer securities to non-accredited investors

A. Actively manage its portfolio A unit investment trust does not have an actively managed portfolio. The securities are selected based on the trust's investment objective and held to the termination of the trust. Investors in the trust must receive a prospectus. Units may be sole to non-accredited investors

FINRA Rules 5130 prohibits the sale of a new issue to all of the following accounts EXCEPT A. An individual account owned by a client of the managing underwrite B. An individual account owned by an employee of another Broker-Dealer C. An individual account owned by an attorney of the managing underwriter D. An individual account owned by the spouse of a Registered Representative

A. An individual account owned by a client of the managing underwrite FINRA Rule 5130 defines a restricted person as any FINRA member firm, a broker-dealer and its employees, finders or fiduciaries (i.e. attorneys and accountants) of the managing underwriter, or immediate family members of any of those persons. New issues may not be sold into an account in which a restricted person has ownership. New issues can be sold to clients of the underwriter

Which of the following activities does not occur during the "pre-registration" period? A. IOIs are confirmed and shares allocated B. Prospectus is prepared C. Due Diligence is performed D. Bake-off

A. IOIs are confirmed and shares allocated Indications of Interest (IOI) are confirmed during the "post-effective" period, once the registration statement has been cleared by the SEC

Which of the following is not regulated by the MSRB A. An issuer of municipal bonds and notes B. A bank dealer that is a member of the MSRB C. A firm that sells municipal securities to institutional clients only D. A brokerage firm that sells only municipal securities

A. An issuer of municipal bonds and notes Brokerage firms and bank municipal dealers, along with all market participants involved in the sale of municipal securities, are subject to provisions of the MSRB. Issuers of municipal securities are not

When must a broker-dealer or its registered representatives obtain suitability information from customers? A. Before execution of each recommended transaction B. Before the recommendation is made C. Any time the client's status changes D. At the time they become clients

A. Before execution of each recommended transaction Suitability must be determined before execution of any recommended transaction. Unsolicited transaction (those initiated by the client on her own) do not require suitability analysis

A hedge fund in which the fund manager is given complete authority to decide on the assets in which to invest is a A. Blind pool fund B. Full discretion fund C. Absolute authorization fund D. Blank check fund

A. Blind pool fund In a blind pool hedge fund, there is not disclosure of the investments that will be made

In a banker:s acceptance (BA) who is liable for paying promised interest and repaying principal? A. Both the bank and the borrower B. Only the bank C. Neither the bank nor the borrower D. Only the borrower

A. Both the bank and the borrower Both the borrower and bank are each liable for paying promised interest and repaying principal at maturity of a BA

Puts or calls of the same issuer share the same A. Class B. Series C. Type D. Sub group

A. Class Puts or calls of the same issuer are part of the same class

An entity engage in oil and gas drilling. Rather than paying income tax at the entity level, it flows through the tax consequences to its investors. It is a: A. Direct Participation Program B. Shell Company C. Syndicate D. Master Trust

A. Direct Participation Program A DPP is characterized by the flow-through of tax consequences to investors It may participate in oil and gas, real estate, agriculture and other types of activities

A member bank of the federal reserve system borrows from the Federal Reserve Bank at the A. Discount Rate B. LIBOR Rate C. Prime Rate D. Fed Funds rate

A. Discount Rate The discount rate is charged by the Federal Reserve when funds are loaned to member banks

Of the Interest rates listed below, which is considered the most volatile? A. Federal funds rate B. Discount Rate C. Prime Rate D. Broker's Call Loan Rate

A. Federal funds rate The Federal Funds rate is the highly volatile rate at which member banks with excess reserves loan to other member banks for the purpose of meeting reserve requirements. It typically changes daily. Also, you should note that although the Fed has a target Fed Funds Rate, it does not actually set the rate. The Fed sets the Discount Rate

Which two of the following options can be combined to create a Protective Put? I. Long Stock II. Long Call III. Long Put IV. Short Call V. Short Stock A. I & III B. II & V C. I & IV D. IV & V

A. I & III A protective put is used to hedge a long stock position by using a put as downside protection. In a worst case scenario The holder of the put sells at the strike price by exercising the put option which maintains the maximum loss always to be the Premium + (Initial Stock Price - Strike Pirce). The loss when using this hedge can never be greater than that amount

When a firm "positions trades," it I. makes a market in securities II. trades for the firm's account III. sells short in all transactions IV. executes agency trades for customers A. I and II only B. I, II, III, and IV C. I, II and IV only D. II and III only

A. I and II only Position trading is another term for a firm making a market in securities and trading for its own account to make a profit

A bond that was yielding 3.65% yesterday is currently yielding 3.75%. Which two of the following statements are true? I. The bond's price went down since yesterday II. The bond's price went up since yesterday III. The bond's yield is 10 basis points more today IV. The Bonds yield is 1/10th basis point more today A. I and III B. II and III C. II and IV D. I and IV

A. I and III There is an inverse relationship between bond prices and bond yields Because the bond's yield is more today, its price fell Basis points measure the change in interest rate. Each basis point equals 1/100 of a percent (A change of 1 percent - 100 basis points). A change from 3.65% to 3.75% is a change of 10 basis points

The compensation to a hedge fund manager is typically I. 1-2% of the assets under management II. 5% of the assets under management III. 10% of the fund profits IV. 20% of the fund profits A. I and IV B. II and IV C. I and III D. II and III

A. I and IV Hedge fund managers generally charge a management fee of 1-2% of the assets under management plus 20% of the fund's profits

Which of the following investment opportunities offer a flow through of losses to investors? I. Closed end funds II. Direct Participation Programs III. Hedge Funds IV. ETFs A. II and III B. I and IV C. II and IV D. I and III

A. II and III Both direct participation and hedge funds offer a flow through of losses that can be used to offset passive income. Limited partnerships are a common form of a direct participation program that provide this tax feature

What vesting schedule does ERISA require for an employee's own contributions to the plan? A. Immediate B. Cliff C. Either graded or cliff D. Graded

A. Immediate ERISA requires that all of the employee's own contributions to the plan must be immediately vested

The 5% policy applies to transactions A. In both listed and unlisted securities B. Executed on a principal basis but not on an agency basis C. In listed securities only D. In securities where a bona fide market is not deemed to exist

A. In both listed and unlisted securities The 5% Policy applies to transactions in both 'listed' and 'unlisted' securities

A closed-end fund investor that is most concerned with issuer risk in an income portfolio should feel most comfortable with which of the following funds? A. Investment-grade corporate bond fund B. A global income fund C. An emerging market debt fund D. A high yield bond fund

A. Investment grade corporate bond fund Of these choices, the investment grade corporate bond fund is most focused on maintaining a portfolio of highly rated bonds that reduce issuer risk. An investment grade rating means that outside agencies have reviewed the issuer's strength and have identified a low risk of default

Which of the following does not describe a function of FINRA? A. It can discipline members who violate its rules, including levying of prison sentences B. Creates rules for the securities industry C. Imposes ongoing educational requirements on its members D. It is funded through assessments on its members

A. It can discipline members who violate its rules, including levying of prison sentences FINRA is a self-regulatory organization (SRO). It is responsible for the oversight of its members through a series of rules and regulations that is creates and enforces. FINRA is funded through fees, dues, and assessments that it imposes on its members. It is not authorized to impose jail time on its members, as FINRA is not a court and has no judicial power.

Which of the following statements is true regarding the underwriting syndicate? A. It is a temporary group that is formed to distribute an offering B. All members of the underwriting syndicate act in an agent capacity C. All firms within the syndicate must be FINRA members D. The underwriting agreement specifies the terms by which all members of the syndicate are to abide

A. It is a temporary group that is formed to distribute an offering The underwriting syndicate is formed to distribute an offering, and disbands after the distribution. The agreement that specifies the terms is the agreement among the underwriters. Members of the syndicate frequently act in a principal capacity as they purchase a share of the offering for their inventories. In corporate underwritings, syndicate members must be FINRA members, but in offering of exempt securities, FINRA membership is not required

An investor expects an advance in the technology sector and would like to take an aggressive position that could provide a great deal of leverage. Which of the following would accomplish this objective? A. Long Index Call B. Long Index Put C. Short Index Call D. Short Index Put

A. Long Index Call Through buying a call on a technology index, an investor establishes a leveraged position that will benefit from an advancing market. A substantial amount of market upside is needed to recover the cost of the premium paid

What is the settlement date that applies to the exercise of index options? A. The business day following the date of exercise B. Two business days following the date of exercise C. Three business days following the date of exercise C. Four business days following the date of exercise

A. The business day following the date of exercise

Which of the following refers to the residual profit after all of a company's expenses have been netted out? A. Net income B. Gross profit C. Sales D. EBITDA

A. Net income Net income is the profit after all of a company's expenses have been subtracted from revenues. It represents the earnings available to shareholders after all obligations (eg debt, payable to vendors, etc.) have been paid. EBITDA is a widely used proxy for operating cash flow as it reflects the company's total cash operating costs for producing its products and services. Gross Profit is defined as sales less cost of goods products and services. Gross Profit is defined as sales less cost of goods sold (COGS). Sales is the first line item on an income statement

All of the following activities have a positive effect on the US balance of payments EXCEPT: A. New US Investment in foreign countries B. Foreign purchases of US securities C. An increase in foreign investments in the US D. An increase in US exports to foreign countries

A. New US Investment in foreign countries New US investments abroad would increase the deficit in the US balance of payments because dollars are leaving the US for investment invested in foreign countries

For a direct participation program, what is the consequence of being a tax pass-through entity? A. No tax at the partnership level B. Taxed at partnership rates C. Taxed at corporate rates D. Qualifies for a partnership exemption

A. No tax at the partnership level When organized as limited partnerships, DPPs are pass-through tax entities. They do not pay income tax at the entity or partnerships level. All profits, losses and tax consequences are passed through to the partners

The organization that maintains a list of suspicious persons and checks customer identification information against known terrorist lists to identify potential money launderers is A. OFAC B. FBI C. FinCEN D. BSA

A. OFAC The Office of Foreign Assets Control (OFAC), a division of the US Treasury, maintains and monitors lists of known terrorists. Firms are required to collect identification information from customers to be checked against the OFAC lists when accounts are opened

The underwriting spread is defined as the difference between the A. POP and the underwriting proceeds B. Underwriting fee and the manager's fee C. Manager's fee and the selling concession D. Total takedown and the underwriting proceeds

A. POP and the underwriting proceeds The underwriting spread is the difference between the price paid to the issuer, known as the underwriter proceeds, and the public offering price, or POP. It is divided into the manager's fee, the underwriting fee and the selling concession

Which of the following is Bullish? A. Protective Put B. Covered Call C. Protective Call

A. Protective Put

In which of the following events will an open order price not be adjusted? A. Reverse split B. Dividend payable in both cash and securities C. Forward stock split D. Stock dividend

A. Reverse split Open orders must be cancelled (not adjusted) at or before the close of business on the day before the ex-date for reverse splits

Which of the following orders will not be adjusted on the ex-dividend 'date? A. Sell limit B. Buy limit C. Sell stop D. Sell stop limit

A. Sell Limit Orders entered below the market will be adjusted on the ex-dividend date. Sell limit orders are placed above the current market

A co-owner in a joint account requests a cash distribution from the account. The broker-dealer must do which of the following? A. Send the distribution as requested and make it payable to both of the joint owners B. Explain that the distribution cannot be sent unless request in writing C. Explain that the distribution can only be sent upon written request signed by both joint owners D. Send the distribution as requested and make it payable to the owner who made the request

A. Send the distribution as requested and make it payable to both of the joint owners A distribution from a joint account may be request by only one of the account owners, but must be made payable to all owners

Which of the following is not a "tool" of the FRB in monetary policy? A. Taxing and spending B. Margin requirements C. Discount rate D. Open-market operations

A. Taxing and spending Taxing and spending is a component of fiscal policy, not monetary policy

Which of the following events is not covered by SEC Rule 145 A. The Declaration of a Cash or Stock Dividend B. Transfers of Assets C. Mergers & Acquisitions D. Reclassifications of Ownership Structure

A. The Declaration of a Cash or Stock Dividend SEC Rule 145 does not address corporate dividend procedures

All of the following statements about municipal revenue bonds are TRUE EXCEPT A. The maturity of the revenue bond usually coincides with the useful life of the facility being built B. They are not subject to debt limitations established by the issuer C. The interest and principal are paid from the revenue received from the facility D. They can be issued by states, political subdivisions, interstate authorities, and intrastate authorities'

A. The maturity of the revenue bond usually coincides with the useful life of the facility being built Municipal Revenue bonds are issued to support long-term infrastructure projects. Bonds that are issued to build them usually mature before the facility is no longer of use. Municipal revenue bonds do not have debt limitations like general obligation bonds. Revenue bongs can be issued by states, political subdivisions (such as counties or townships), interstate authorities and intrastate authorities. The interest and principal owed to bondholders is paid from the revenue received from the facility

All of the following statements about variable annuity surrender charges are true EXCEPT A. They typically increase each year of the surrender charge period B. They are in addition to other mortality, expense and administrative charges that apply to variable annuity contracts C. They often apply for the first 6-10 years of the contract D. They are charged to protect the insurance company from early withdrawals of invested funds

A. They typically increase each year of the surrender charge period Surrender charges typically decrease over a surrender charge period. They are assessed as a percentage of the amount of the payment withdrawn

A UIT sells units to investors A. Through representatives of broker dealers with which selling agreements have been established B. Through the trustee that administers the trust C. In private placements only D. Directly through the sponsor

A. Through representatives of broker dealers with which selling agreements have been established The units of UITs are typically distributed by the representatives of broker dealers. A broker dealer must have a selling agreement with the UIT to distribute the UIT:s shares through its representatives

Why do companies typically participate in *reverse-split* corporate events A. To increase a low price per share B. To pay a special dividend C. To deter hostile takeovers D. To reward stockholder

A. To increase a low price per share Reverse splits are a way of increasing low share prices. This can be useful in some cases in avoiding exchange de-listing as a result of low share prices

An investor in a high tax bracket is subject to federal, state and local income taxes. If the investor is seeking current income with minimum tax liability, which of the following choices is most appropriate? A. US Virgin Island Utility bonds B. Double barreled municipal bonds C. Private activity municipal bonds D. Zero Coupon Treasury Strips

A. US Virgin Island Utility bonds Bonds issued by US Virgin Islands and other US territories and possessions are exempt from federal, state, and local taxes. Double barreled bonds, like other municipal bonds are exempt from federal and possibly state taxes Private activity bonds may be subject to alternative minimum taxes Treasury Strips are subject to federal income tax and do not provide current income

All of the following are associated with new issues of revenue bonds EXCEPT A. Voter referendums B. Backing by fees raised from a specific project C. No limitations on taxing authority D. Backing from a private corporation

A. Voter referendums Voter referendums are required in many new issues of general obligation bonds. They are not required for revenue bond. Revenue bonds are backed by user fees, or potentially by fees paid by a private corporation for the use of the facility. Revenue bonds are not subject to limits on the taxing authority of the issuer.

Lisa, age 80, was supposed to take a Required Minimum Distribution of $8,000 for 2020. However, her accountant made a mistake and she only took out $6,000. Must she pay a penalty? A. Yes, $1,000 B. Yes, $500 C. Yes, $2,000 D. No, because the mistake was not intentional

A. Yes, $1,000 She must pay an excise tax of 50% on the under-distribution. Her under-distribution is $2,000 and the excise tax is $1,000

All of the following statements are true about convertible bonds EXCEPT A. a bond cannot be both callable and convertible B. convertible bonds help issuers achieve lower fixed costs for borrowing C. the bond indenture explains the terms under which conversion can occur D. the conversion ratio specifies the number of shares the bond holder will receive per bond exchanged

A. a bond cannot be both callable and convertible Issuers may add call provisions to convertible bonds. The bond indenture specifies the terms under which a bond can be converted. The conversion ratio specifies the number of shares a bond holder will receive per bond exchanged; the conversion price is the price of the underlying stock at which the bond can be converted to common. Because convertible bonds offer a choice to bondholders, issuers can pay a lower rate, and reduce their fixed costs for borrowing money from investors

A six-month decline in business activity, employment and stock price is A. a recession B. a depression C. stagflation D. inflation

A. a recession A recession is defined as 2 or more consecutive quarters of negative GDP growth. Declines in business activity, employment and stock prices all indicated a recession

a bond with a 5.1% coupon is trading at $942. It has a current yield of 5.4%. The bond's yield to maturity is A. above 5.4% B. below 5.1% C. exactly 5.4% D. between 5.1% and 5.4%

A. above 5.4% For a discount bond (trading below par value), a bond's yield to maturity (YTM) will always be greater than current yield

The tax benefits of a municipal bond apply to A. interest income generated from the bond B. any capital gain from the sale of the bond C. individual investors only, not institutions D. both capital gains and interest income generated from the bond

A. interest income generated from the bond The tax benefits of a municipal bond apply to the bond's interest income only, not any capital gains that may be realized

In a disciplinary action against a registered representative, FINRA may impose any of the following EXCEPT A. sentence the rep to two years in jail for defrauding a customer B. bar the representative from association with a member firm C. cancel the representative's securities registration D. fine the representative $25,000

A. sentence the rep to two years in jail for defrauding a customer As an enforcer of securities industry regulations, FINRA has the authority to censure, bar, suspend, cancel registration or membership, and levy fines. FINRA cannot impose a prison sentence, but could refer a case to the attorney general for criminal proceedings

All of the following are permitted activities, EXCEPT A. short tendering stock B. overselling an IPO and subsequently repurchasing the shares in the secondary market, at a profit C. publishing favorable research on a company's bonds just prior to the effective date of an equity follow-on offering D. stabilizing an IPO at a price at or below the public offering price

A. short tendering stock During a tender offer, an investor can tender only up tp the amount of their net long position. An investor could not tender more than their net long position. Stabilizing, overselling a new issue and publishing research on unrelated products are all permitted

The time value of an option contract represents A. the amount of the premium in excess of the intrinsic value B. the intrinsic value of the contract C. the option premium plus the intrinsic value D. the out of the money amount of the contract

A. the amount of the premium in excess of the intrinsic value Time value represents the amount of an option premium in excess of its intrinsic value. For option contracts which carry no intrinsic value (out of the money positions), the entire premium is time value

All of the following statements are true of a callable convertible preferred issue EXCEPT that A. the stockholder must surrender preferred when called or lose the right to par value B. the convertible preferred can increase in price because of the underlying security C. the convertible is issued with a lower stated dividend rate than a nonconvertible preferred D. dividend payments stop after the preferred is called

A. the stockholder must surrender preferred when called or lose the right to par value The advantage of convertible preferred stock is the opportunity to participate in any appreciation in the value of the common stock. In return for this potential benefit, the preferred pays a lower rate than nonconvertible preferred. Stockholders are always required to surrender the stock when it is called, but they will usually be paid a premium over par value. Additionally, dividends cease on the call date. Note that this is an "except" question

Shares of ABC Co., a NYSE listed company, are traded over-the-counter. The trade is said to take place in the A. third market B. secondary market C. fourth market D. primary market

A. third market The third market is when exchange-listed securities trade over-the-counter. The primary market is when a company sells new shares to raise capital. The secondary market is investor to investor trading of those shares. The fourth market is trades conducted via electronic communication networks

Miss Jones purchased a 7% J&J 15 municipal bond on Tuesday, March 18th in a regular way trade. How much-accrued interest will she pay? A. $15.36 B. $12.64 C. $13.61 D. $16.13

B. $12.64 Figure out the number of days of accrued interest between the most recent interest payment date and the settlement date. Municipal bonds trade regular way, T+2, so the settlement date is Thursday, March 20th. They also trade on a 30/360 basis, so there are 16 days of accrued interest in January (15-30th, including the 15th), 30 days in February, and 19 days in march (do not include settlement date), for 65 days of accrued interest. To calculate the accrued interest payable: 65 days/360 days per year x $70 (annual interest) = $12.64 in accrued interest

When executing transaction in municipal securities for or on behalf of a customer as agent, firms must make an effort to obtain a price for the customer that is A. A price that is no higher than that charged by other municipal securities firms B. A fair and reasonable price to prevailing market conditions C. Sufficient to ensure profit to the firm yet is less than a 3% commission D. The best price currently available

B. A fair and reasonable price to prevailing market conditions When executing a transaction in municipal securities for or on behalf of a customer as agent, firms must make an effort to obtain a price for the customer that is fair and reasonable in relation to prevailing market conditions

A new issue of municipal bonds has an aggregate par value of $12 million. The following syndicate orders have been received: $5 Million in group orders; $5 Million in member orders; $5 Million is designated orders. What is the allocation priority of these orders from highest to lowest? A. $5 million in member orders; $5 million in designated orders; $2 million in group orders B. $5 million in group orders; $5 million in designated orders; $2 million in member orders C. $5 million in group orders; $5 million in member orders; $2 million in designated orders D. $5 million in designated orders; $5 million in member orders; $5 million in group orders

B. $5 million in group orders; $5 million in designated orders; $2 million in member orders The allocation priority is found in the syndicate letters, and usually follows the following priority: Presale, Group net, Designated, Member. Only $2 million of the member orders can be filled

An investor buys $1 million of 10% corporate bonds at par. At the end of the day, the bonds close up 1/2 point. The investor has a gain of A. $2,500 B. $5,000 C. $50,000 D. $25,000

B. $5,000 An Increase of 1/2 point represents an increase of .5% or .005 x 1,000,000 = $5,000

What is the minimum cooling-off period for a public securities offering? A. 60 days B. 20 days C. 10 days D. 30 days

B. 20 days The cooling-off period begins with the filing of a registration (S-1) with the SEC. It then lasts at least 20 days, but in any case until the SEC has approved the registration

Commercial paper that is exempt from registration requirements has a maximum maturity of how many days? A. 360 B. 270 C. 180 D. 90

B. 270 Commercial paper with a maximum maturity of 270 days is exempt from registration. Municipal bonds, government bonds, and domestic bank securities are also exempt from registration

The overall objective of asset allocation is to ensure that investors have A. An equally balanced portfolio of stocks, bonds, and cash B. A blended portfolio of assets that will react differently under different market conditions C. A portfolio of securities tailored to their specific goals D. Sufficient cash available to make investments that are recommended by prominent investment managers

B. A blended portfolio of assets that will react differently under different market conditions The objective of asset allocation is to ensure that the portfolio includes asset categories that react differently under diverse market conditions

What is a repurchase agreement or repo? A. A contractual arrangement between a customer and a broker-dealer, in which the broker-dealer gives a selling customer the right to repurchase sold securities for 30 days B. A contractual arrangement between two parties, in which one party agrees to sell securities to another party at a specified price with a commitment to buy the securities back at a later date for another specified price C. A contractual arrangement between two parties, in which the broker-dealer can repossess securities pledged as collateral in connection with margin loans D. A contractual arrangement between a customer and a broker-dealer, in which the broker-dealer gives a selling customer the right to repurchase sold securities up until the settlement date

B. A contractual arrangement between two parties, in which one party agrees to sell securities to another party at a specified price with a commitment to buy the securities back at a later date for another specified price A repo is a contractual agreement between two parties, in which one party (a borrower of cash) agrees to sell securities to another party (lender of cash) at a specified price with a commitment to buy the securities bak at a later date for another specified (typically higher) price. This higher price reflects the interest earned by the purchaser (lender)

Which type of business is the most likely the target of a private equity firm? A. A large public company that has been mismanaged B. A private company that is underperforming C. A publicly trade limited partnership that needs additional capital D. A small public company that is undervalued

B. A private company that is underperforming Private equity funds target underperforming private companies with the intent of turning them into strong performers. They receive their payoff by taking the company public through an IPO

All of the following practices are prohibited in the sales of mutual fund shares EXCEPT A. A redemption request that is received today is processed at the NAV calculated at the previous day's close B. A registered representative recommends that an investor make a slightly larger investment to qualify for a breakpoint C. An investor is encouraged to purchase shares in a fund just before the ex-dividend date to receive the dividend distribution D. An investor is encouraged to engage in trading strategy that involves regularly purchasing and redeeming shares within a short time frame to take advantage of share pricing inconsistencies

B. A registered representative recommends that an investor make a slightly larger investment to qualify for a breakpoint Breakpoints allow investors to receive a discount on the sales charge based on the dollar amount invest. Registered Representatives are required to disclose the existence of breakpoint to clients. Late trading is the prohibited practice of redeeming or purchasing shares at a price previously calculated instead of following the forward pricing rule. Market timing, or executing short term purchase and sales of mutual fund shares is also a prohibited practice. Because of their fee structures, mutual funds should be recommended as long-term investment. Encouraging the purchase of shares prior to a dividend distribution is called "selling dividends" and subjects the investor to a taxable event and a reduction in share value, since the NAV of fund shares falls when a dividend is distributed.

An individual would receive an annual report of a corporation:s recent results if they are A. An attorney B. A stockholder C. A bondholder D. A creditor

B. A stockholder As a common stockholder an individual would be entitled to receive reports directly from the corporation

A share in a foreign corporation that underlies an ADR is known as a(n) A. GDS B. ADS C. ADP D. GDR

B. ADS An American Depositary Share (ADS) is the name given to the shares of the foreign corporation that are held by the depositary institution. These shares are packaged to create the ownership interests known as American Depositary Receipts. A Global Depositary Share (GDS) is the underlying for a Global Depositary Receipt

In regard to a bond's tax cost basis, what is the main difference between accretion and amortization? A. Accretion applies to government bonds; amortization to corporate and municipal bonds B. Accretion applies to bonds bought at a discount; amortization to bonds bought at a premium C. They are synonymous terms; there is no difference D. Accretion applies to long term bonds; amortization to short term bonds

B. Accretion applies to bonds bought at a discount; amortization to bonds bought at a premium Accretion is the upward adjustment towards par of the cost basis of a bond purchased at a discount. Amortization is the downward adjustment towards par of the cost basis of a bond purchased at a premium

All of the following statements about emergency contacts for a broker-dealer:s business continuity plan are true EXCEPT A. Emergency contacts for the business continuity plan must be reported to FINRA B. All emergency contacts must be registered principals C. One of the emergency contacts must be a member of the firms senior management D. The firm must name two emergency contacts

B. All emergency contacts must be registered principals A firm is required to name two emergency contacts for its business continuity plan and report this information too FINRA. One of the persons must be a registered principal and a member of senior management of the firm

When must a broker-dealer ensure that a recommended security is suitable for its customer? A. As long as the customer stays with the firm B. At the point of sale C. For the life of the investment D. At the point of sale and for one year following the sale

B. At the point of sale Broker-dealers must apply suitability and fair-dealing standards and rules at the point of sale. The sale can be a recommendation to purchase, hold, or sell a security. However, broker-dealers and their representatives usually do not have responsibilities to monitor customers' investments beyond the point of sale

All of the following statements are true regarding bond trading markets EXCEPT A. The counterparty to a bond trade is typically a bonk or dealer B. Bonds are usually traded at a centralized location or exchange C. Bonds are bought and traded mostly by institutions such as pension funds, banks and insurance companies D. The compensation for a bond trade is typically not commission but the spread between bid and ask price

B. Bonds are usually traded at a centralized location or exchange Unlike equity trading, Bond trading usually takes place between dealers in decentralized over-the-counter markets. Bonds are typically bought and sold by institutions; individuals are more likely to purchase bond funds. The spread between bid and offer price is typically the compensation for a bond trade, while equity trades are typically compensated by brokerage commissions

Dividend and capital gains distributions are made to investors in A. Unit investment trusts but not open-end funds B. Both open-end funds and unit investment trusts C. Neither open-end funds nor unit investment trusts D. Open-end funds but not unit investment trusts

B. Both open-end funds and unit investment trusts These distributions are made for both open-end funds and unit investment trusts. These are among the benefits of investing in these products

The pricing characteristics of REIT shares are most like those of shares of a: A. Growth mutual fund B. Closed end fund C. Money market fund D. Direct participation program

B. Closed end fund REIT shares trade in the secondary market at a discount or premium to their NAV like closed end fund shares. Mutual fund shares like growth funds or money market funds are redeemed by the fund at their NAVs DPPs do not generally trade, there is an extremely limited market for investors who wish to sell their interests

An investor with the goal of income preservation would be most likely to purchase A. 1,000 shares of common stock B. Commercial paper and T Bills C. Direct Participation programs D. Exchange traded funds

B. Commercial paper and T Bills An individual with a goal of income preservation would be most likely to purchase a money market security, such as commercial paper or Treasury paper

Which of the following can be used to enhance the return on stock positions? A. Protective Put B. Covered Call C. Protective Call

B. Covered Call

Which of the following is is market neutral, but slightly bullish? A. Protective Put B. Covered Call C. Protective Call

B. Covered Call

If a time in force order instruction is not given, the default usually is... A. Good-til-date order B. Day order C. Immediate-or-cancel order D. Good-til-cancelled order

B. Day order A day order remains in effect only for the duration of the trading day on which it is entered. It generally is the default unless another time in force instruction is entered

Organizations that have regulatory oversight over variable products include all of the following EXCEPT A. FINRA B. FDIC C. State Insurance Commissioners D. SEC

B. FDIC The FDIC is a banking regulator and does not have regulatory authority over variable products. Securities regulators and insurance regulators are charged with regulatory oversight of variable products. These regulators include the SEC, FINRA, and state insurance commissioners.

The inputs for calculating yield to call (YTC) include the bond's current market price, par value, and what else? A. Maturity date and call premium B. First call date and any call premium C. First call date D. Maturity date

B. First call date and any call premium YTC is calculated the same as yield to maturity (YTM) with two expectations. First, the first call date is used instead of maturity date Second, any call premium on the first call date must be added to par value

Which of the following is NOT primary responsibility of broker-dealer's transfer agent? A. Issues and cancels certificates on behalf of the issuer B. Guarantees the signature of the parties involved in the transfer C. Acts as the intermediary for the issuer D. Validates mutilated certificates and assists in tracking lost or missing securities

B. Guarantees the signature of the parties involved in the transfer The transfer agent reviews all information for accuracy and completeness. It checks to ensure signatures are authentic, but does not guarantee them. The guarantee of the signature is handled by the broker-dealer through the Medallion Stamp Process

Which two of the following are bullish? I. Long Call II. Long Put III. Short Call IV. Short Put A. I & II B. I & IV C. II & III D. II & IV

B. I & IV

How do ETFs differ from closed-end company shares I. ETFs are typically higher cost investments II. ETFs are typically lower cost investments III. ETFs are typically more actively managed IV. ETFs are typically less actively managed A. I and IV B. II and IV C. I and III D. II and III

B. II and IV ETFs are often less costly to own than closed-end company shares because they employ a more passive fund management strategy. Their portfolios are often constructed to replicate a major index or other market benchmark and less actually managed

Which two of the following statements are TRUE regarding the MSRB gift rule? I. The annual gift limit is $100 II. Occasional gifts of meals or tickets to theatrical or sporting events are permitted even if they are in excess of the limit III. Gifts that include logos may be given without regard to value IV. Employment contracts between firms are prohibited by this rule A. II and III B. I and II C. II and IV D. I and III

B. I and II The MSRB annual gift limit is $100 per year for gifts given in connection with the business of the employer of the recipient. Exceptions are made for occasional gifts of tickets or meals. Deductible business expenses and employment contract are not subject to this limit. Logo gifts or reminder advertising may be given if they are of reasonable value

Which of the following apply to sale of variable insurance products? I. FINRA Suitability requirements II. The Know Your Customer Rule III. Anti-Fraud Provisions of the Securities Act of 1934 A. II and III only B. I, II, and III C. I and III only D. I and II only

B. I, II, and III Variable products are securities because customers may lose money. They are subject to FINRA rules of suitability and the Know Your Customer rule which requires the collection of sufficient information to understand investment objectives. The anti-fraud provisions of the Securities Exchange Act of 1934 apply to the sale of all securities.

Which two of the following options strategies are bearish? I. Long Call II. Short Call III. Long Put IV. Short Put A. I & II B. II & III C. III & IV D. I & IV

B. II & III A bearish investor profits when market prices fall. Call writers and put buyers are bearish. Put buyers have the right to sell at the exercise price in a declining market, while call sellers are not assigned when market prices fall. They may keep the premium received without having to sell stock.

In June, an investor purchases a December XYZ 60 call premium of $500. By September the premium is $800. if the investor directs his broker to close out the transaction, which two of the following are TRUE? I. A closing purchase will take place II. A closing sale will take place III. The investor will realize a profit of $300 IV. The investor will realize a loss of $300 A. I & III B. II & III C. I & IV D. II & IV

B. II & III An investor that purchases an option can close the transaction to realize a profit prior to expiration by selling the option when the premium is higher. To close out a long option position, a closing sale transaction is executed. The Investor will realize a profit because the option is sold for more than the purchase price $500.

Which two of the following options can be combined to create a Protective Call? I. Long Stock II. Long Call III. Long Put IV. Short Call V. Short Stock A. I & III B. II & V C. I & IV D. IV & V

B. II & V A Protective call is made up of a short stock position with a long call to hedge. This combination allows for the speculation on downward movement of a stock (short stock) with protection on appreciation (long call).

A 4% municipal bond is trading in the secondary market for 96. Which two of the following statements about the bond's current yield are true? I. The bond's current yield is 4% II. The bond's current yield is 4.17% III. The bond's CY is less than its YTM IV. The bond's CY is greater than its YTM A. II and IV B. II and III C. I and IV D. I and III

B. II and III Current yield is calculated by dividing the annual interest rate of the bond 4% by the current price 96. Because the bond is trading at a discount its current yield is greater than the coupon rate, and its YTM is greater than its current yield.

An investor has purchased a European style put option on the S&P 500 Index. Which two of the following statements are TRUE? I. The contract can be traded only on exchanges outside of the US II. At exercise, the holder will receive cash III. The contract may only be exercised on the expiration date IV. The contract may be exercised any time after purchase but prior to expiration A. II and IV B. II and III C. I and III D> I and IV

B. II and III Index options contracts are both American and European style. Options on stock, or equity options, are American style contracts only. American style options may be exercised at any time after purchase but before the contract's expiration date. European style options may only be exercised at expiration. Index options settle in cash

Which TWO of the following order would NOT be reduced on the ex-dividend date? I. Buy limit order II. Buy stop order III. Open sell stop order IV. Sell limit order A. I and III B. II and IV C. I and IV D. II and III

B. II and IV All order placed below the market are adjusted downward on the ex-dividend date by the amount of the dividend. Orders placed below the market are buy limits, sell stops, and sell stop limits. Sell limits, buy stops and buy stop limits are not adjusted on the ex-dividend date

A broker-dealer is acting as a trustee for an issuer, holding ownership information of that issuer's common stock. The broker-dealer may use this ownership information to solicit additional business for itself A. If the security owners are notified in writing of this relationship B. If the issuer specifically consents to this C. Under no circumstances D. If the issuer monitors how the information is used

B. If the issuer specifically consents to this Issuer consent would be required in this type of situation

Investors that purchase Ginnie Mae, Fannie Mae or Freddie Mac mortgage-backed securities will benefit from which of the following? A. Interest payments every six month B. Income that is slightly higher than income from US Treasury securities C. Full backing by the US Government D. Exemptions from interest taxation at the federal, state and local level

B. Income that is slightly higher than income from US Treasury securities Investors purchase mrtgage-backed securities for the monthly income stream that is very safe from default. Should these issuers ever default, the government would probably use its creditworthiness to guarantee investors' payments of interest and principal. Some agency securities receive exemptions from state and local taxation, but mortgage-backed agency securities are subject to taxation at all levels

Equity REITS offer the potential for all of the following EXCEPT A. Market Transparency B. Interest income paid on a monthly basis C. Liquidity D. Inflation hedging

B. Interest income paid on a monthly basis Equity REITs do not produce interest income. They pay dividends to investors that represent their proportionate share of the rental income and capital appreciation earned by the portfolio. Because they trade on exchanges they offer liquidity and transparency. Real estate is also a natural hedge against inflation, and has historically appreciated at a rate that exceeds the rate of inflation.

All of the following are provisions of the Trust Indenture Act of 1939 EXCEPT A. If a bond issuer becomes insolvent, the appointed trustee may be given the right to seize the bond issuer's assets and sell them in order to recoup the bondholder's investments B. It applies to the issues of Corporate debt securities in excess of $1 million C. Issuers must name a trust corporation to carry out the terms of the indenture D. Issuers are required to create a written document that discloses the terms and conditions under which the securities are issued, including legal obligations and any restrictions

B. It applies to the issues of Corporate debt securities in excess of $1 million The Trust Indenture Act of 1939 applies to corporate issues of $50 million and above, and requires a formal written agreement (an indenture), signed by both the bond issuer and the bondholder, that fully discloses the particulars of the bond issue. The act also requires that a trustee be appointed for all bond issues, so that the rights of bondholders are not compromised

The theory that says the economy is best controlled through taxation and government spending is known as A. Classical economic theory B. Keynesian economic theory C. Monetarist economic theory D. Open market operations

B. Keynesian economic theory This is the framework behind Keynesian economic theory, founded by John Maynard Keynes in the 1930s

A speculative investor has a strong bearish outlook on ABC stock. Which of the following positions is most suitable for this invest? A. Long ABC Call B. Long ABC Put C. Short ABC Call D. Short ABC Put

B. Long ABC Put A long put is the most bearish of the options strategies. When not used to protect a stock position it allows an investor to benefit from a declining market.

An investor anticipates a decline in the broad market and expects profit potential based on the degree of the decline. This investor's position is which of the following? A. Long Index Call B. Long Index Put C. Short Index Call D. Short Index Put

B. Long Index Put An investor that is bearish on the overall market benefits by buying index puts. There must be a substantial decline in the stock to profit, because the investor must recover the premium paid.

An investor that owns 100 shares of ABC stock wishes to protect his position as much as possible from a potential market correction. Which of the following positions provides the best protection. A. Long ABC Call B. Long ABC Put C. Short ABC Call D. Short ABC Put

B. Long an ABC Put An investor that owns stock can best protect it from downside risk by purchasing a put. The put gives the investor the right to sell at the exercise price. A short call also provides some protection but only in the amount of the premium received.

An investor should expect the greatest price increase in which of the following if interest rates decline? A. Short-term bonds selling at a discount B. Long-term bonds selling at a discount C. Short-term bonds selling at a premium D. Long-term bonds selling at a premium

B. Long-term bonds selling at a discount Because of the inverse relationship between price and yield, when interest rates fall, bond prices rise. Longer maturities have more market risk, so their prices rise more than shorter maturities. Bonds selling at a discount also rise more sharply than those selling at a premium

According to Federal law and industry regulations, a firm must deliver a prospectus A. Only for sales of mutual funds or other continuous primary offering securities B. No later than with the confirmation of sale of newly issued securities C. Prior to offering new securities for sale D. At or before a trade is entered for newly issued securities

B. No later than with the confirmation of sale of newly issued securities The Securities Act of 1933 required that a prospectus is given to all purchasers of new securities no later than with the confirmation of sale

Which organization guarantees the performance of standardized options contracts? A. SEC B. OCC C. FINRA D. NORA

B. OCC The Options Clearing Corporation (OCC) is the world's largest equity derivatives clearing house. As a clearinghouse, the OCC also acts as a guarantor, ensuring that the obligations of the contracts it clears are fulfilled.

The document that most comprehensively discloses municipal bond and issuer information to potential investors is: A. Statement of Additional Information B. Official Statement C. Prospectus D. Bond Indenture

B. Official Statement Like a prospectus for corporate securities, an official statement provides detailed information about the issuer's financial and operating condition and details of bond offering

A type of stock that offers the possibility of higher dividend than the stated rate is A. Cumulative preferred stock B. Participating preferred stock C. Callable preferred stock D. Convertible preferred stock

B. Participating preferred stock Participating preferred stock offers the possibility of a higher dividend than the stated rate

Maud and Louis are an elderly retired couple seeking to preserve their life savings. They attend a free-lunch seminar where many types of securities are discussed. Which of the following is the least suitable investment for Maud and Louis? A. AAA Corporate Bond B. Real estate limited partnerships C. Open-end investment companies D. T Bills

B. Real estate limited partnerships Real estate limited partnerships will offer the least amount of safety and liquidity. Given the shorter investment horizon, lack of current income and goal to preserve principal these investors would likely seek safe, liquid investments and may seek current income to assist in their meeting current expenses

Which of the following events does not raise capital for an issuer? A. Follow-on offering B. Rule 144 Sale C. Shelf Registration D. IPO

B. Rule 144 Sale A Rule 144 sale does not raise capital for an issuer. The rule is used by insiders and affiliates who are seeking to liquidate a portion of their holdings

Which of the following regulatory bodies not an example of a Self-regulatory organization (SRO) A. MSRB B. SEC C. FINRA D. NYSE

B. SEC The SEC is a US government agency that is responsible for enforcing federal securities laws and regulating the securities industry

All of the following are true of both mutual funds and REITs except: A. Both have a Board of Directors to establish their strategic direction B. Secondary Market Trading offers liquidity to investors C. Investors own a proportionate shares of diverse assets that is professionally managed on their behalf D. A prospectus must be made available to potential purchasers

B. Secondary Market Trading offers liquidity to investors Mutual fund shares do not trade on the secondary market They are redeemed by the fund There are both publicly traded, and private, but non-traded REITs

A customer wishes to sell 100 shares of stock that are now trading at $26 per share. Which of the following instructions is valid for a sell stop order? A. Sell 100 shares at $26 stop limit B. Sell 100 shares at $24 stop C. Sell 100 shares at $28 stop D. Sell 100 shares at the market

B. Sell 100 shares at $24 stop Stop orders to buy must be placed above the current market price. Stop order to sell must be placed below the current market price. In this case, if the price drops as low as $24, the stop order will convert to a market order and be immediately executed at the best price

Preferred stock trades most like which of the following instruments? A. Common stock B. Straight debt security C. Convertible bond D. Call option

B. Straight debt security Because of the fixed dividend payment, preferred shares are influenced in the market by the same factors that impact straight debt securities. Like bonds, there is an inverse relationship between price and interest rates

By endorsing ownership of a stock certificate to a brokerage firm, an investor transfers ownership into A. An escrow account B. Street name C. Negotiable name D. An omnibus account

B. Street name Street name ownership means that the stock certificate is negotiable only by the brokerage firm named in the endorsement. The firm maintains a record of ownership and the investor may sell the stock without the need to deliver a paper certificate

When compared to the expenses associated with the purchase of mutual fund shares, the cost of investment in a UIT is A. Less expensive for large purchases only B. Substantially cheaper C. More Expensive D. Usually about the same

B. Substantially cheaper The sales charges for investing in UITs are substantially less than comparable mutual fund investments because of the fixed portfolio. There is no management fee

In interest-bearing corporate bonds, accrued interest that is due the seller must be included included in settlement. According to industry rules, the accrued interest must be calculated up to what date? A. T+2 B. T+1 C. T+3 D. T

B. T+1 For interest bearing fixed income securities, interest is accrues up to (but not including) the settlement date (T+2). Put differently, accrued interest includes the trade date and T+1 but not on T+2 (settlement). On T+2, the interest accrues to the buyer of the bond

Who sets the margin requirement for OTC stocks A. The FINRA B. The FRB C. The NYSE D. The SEC

B. The FRB The FRB sets the margin requirements for OTC stock and decides which OTC stocks are margin-able on a case-by-case basis

The regulation that requires registration of all exchanges and the persons that trade on them is A. The Glass Steagall Act B. The Securities Exchange Act of 1934 C. The Securities Act of 1933 D. The Maloney Act

B. The Securities Exchange Act of 1934 The Securities Exchange Act of 1934 regulates and requires the registrations of exchanges, broker-dealers and their representatives, and other participants in the secondary market

A cash dividend is declared on ABC stock. Which of the following statements regarding outstanding call options on ABC stock is correct? A. The dividend will be paid to the writer of the option B. The dividend will be paid to holders of ABC options only if they exercise the option before the ex-dividend date C. The payment of the dividend is unlikely to impact the trading price of the option D. The dividend will be paid to the holder of the option

B. The dividend will be paid to holders of ABC options only if they exercise the option before the ex-dividend date When csh dividend is declared on stock, whoever owns the stock s of the ex-dividend date receives the dividend while the contract is still trading. However, holders of call options may choose to exercise options early to capture the cash dividend. They must exercise the contract before the ex-dividend date to receive the dividend. When a cash dividend is paid, the stock price generally drops on the ex-date by the amount of the dividend. This change in price of the underlying stock will be reflected in the market price of the option as it continues to trade.

The files of a customer who received a recommendation to purchase an interest in a public oil and gas program must include A. Written approval from the principal approving the recommendation before it was delivered to the customer B. The documents collected to support the determination of suitability of recommended partnership interest C. A signed statement that the customer does not hold the general partner responsible for potential financial loss related to the partnership business D. A signed attestation that the customer has reviewed the risks of the partnership with a purchaser representative

B. The documents collected to support the determination of suitability of recommended partnership interest FINRA rules require that representatives maintain in the files of the customer the documents that support the determination of suitability of the partnership interest for the customer

ABC Law Firm has just changed the rating of the Friendly School District bonds from BBB+ to AA-. As the result, A. The yield on these bonds would rise while their prices would fall B. The price of these bonds would increase while their yields would fall C. School districts in general would be viewed as generally unattractive investments D. The issuer will likely be facing an increase in insurance payments

B. The price of these bonds would increase while their yields would fall The rating of these bonds has increased. The expectation, therefore, would be to see a decrease in yield and an increase in price

With regard to municipal taxing authority, which of the following statements is TRUE? A. Most cities and counties may issue GO bonds without voter referendums B. The taxing authority of municipalities varies widely depending on applicable state or local laws C. A high debt limit is viewed as a greater sign of safety than a low debt limit D. States usually rely on property taxes to back GO bond issues.

B. The taxing authority of municipalities varies widely depending on applicable state or local laws Municipal taxing authority and statutes that address types and amounts of taxes vary considerably. Most local government bond issues require voter approval or referendums before they can be issued. States typically rely on sales or income taxes for backing of bond issues. A low debt limit is typically viewed more favorably because the municipally potentially has less outstanding debt to service

What is the maximum number of accredited investors allowed in a private placement offering? A. 50 B. There is no limit C. 35 D. 300

B. There is no limit There is no limit on the number of ACCREDITED investors in a private placement

When must an elderly person stop making contributions to a Traditional IRA? A. When Social Security begins B. There is not limit C. At normal retirement age D. When IRA distributions begin

B. There is not limit There is no longer an age limit for contributing to a Traditional IRA. This is a rule change under the Secure Act passed in December 2019. Note that there used to be an age limit for opening a Traditional IRA of 70 1/2. Candidates should be familiar with both the new and the old rule. There is not age limit for opening a Roth IRA brokerage account

In which of the following areas are open-end funds an exchange-traded funds similar? A. They both have active secondary markets B. They both make capital gains distributions to investors C. They both have static, or fixed, portfolios D. They both issue redeemable shares to the public

B. They both make capital gains distributions to investors Both open-end funds and exchange-traded funds make capital gains distributions to investors

All of the following statements describing REITs are true EXCEPT: A. They offer ownership in a pool of assets much like a mutual fund B. They generally invest in residential real estate only C. Shares trade in the secondary market at a discount or premium to their net asset value D. Public REITS are first offered to the market through an IPO

B. They generally invest in residential real estate only REITs generally invest their assets in commercial real estate, but also hold residential real estate mortgages. The ownership interest held by an investor reflects a share in all properties held by the fund much like ownership of mutual fund shares. REIT shares trade in the secondary market at a premium or discount to their NAV REITs raise capital through an IPO

All of the following are characteristics of American Depositary Receipts EXCEPT A. They represent ownership in shares of a non-US company B. They trade on exchanges outside of the US C. They may be sponsored or unsponsored D. They pay dividends in US dollars

B. They trade on exchanges outside of the US American Depositary Receipts (ADRs) represent an interest in the shares of a non-US company. They trade in the US financial markets and save investors the inconvenience of cross-border and cross-currency transactions. Dividends are paid in US dollars. They may be issued as sponsored or unsponsored. ADRs are unsponsored if there is no agreement in place with a depositary. ADRs may be issued and serviced by more than one depositary if they are unsponsored

Which of the following economic indicators is considered a lagging indicator? A. GDP B. Unemployment C. Stock market returns D. Retail sales

B. Unemployment The unemployment rate is considered a lagging indicator, because it tends to react several quarters after changes in the economy. Stock market returns are considered a leading indicator; while GDP and retail sales are considered coincident indicators

Two weeks after purchasing a May 80 call option, the value of the underlying equity security is 95. In order to benefit from this situation, the investor A. must exercise the option in order to realize a profit from this position B. can either exercise the option or liquidate the position in the open market C. must allow the option to expire D. must place an order to sell the underlying equity security

B. can either exercise the option or liquidate the position in the open market If an investor owns a call option which is "in-the-money" (has intrinsic value), the investor may choose to either exercise the option, or liquidate the position in the open market

The agreement that specifies rules for the group of financial institutions who assist an underwriter in the sale of a new securities issues but do not have responsibility for any unsold securities is a(n) A. letter of intent B. selected dealer agreement C. agreement among underwriters D. syndicate letter

B. selected dealer agreement A selected dealer agreement is used between the underwriter or syndicate and selling group member. The selling group assists in the selling of new securities without financial responsibility for any of the unsold issue

A municipal bond issue is structured whereby a portion of the issue is retired each year. This is an example of a A. sinking fund B. serial bond C. balloon bond D. term bond

B. serial bond This is an example of a serial bond structure. In this scenario, outstanding bonds are retired at different intervals with a portion of the issue maturing each year

Which type of bonds have interest coupons attached and interest is payable to the individual who represents the appropriate coupon?

Bearer Bonds

Which of the following securities is the least liquid? A. Exchange traded common stock B. Treasury bond C. Hedge funds D. Commercial paper

C. Hedge funds A hedge fund is not a liquid product. It usually requires a significant investment of capital, and a long-term invest horizon

An Investor sells a put option for a premium of $4. The strike price is $120. The Investor's breakeven is A. $124.00 B. $120.00 C. $116.00 D. $128.00

C. $116.00 To calculate breakeven on a put, subtract the premium from the strike price. ($120-$4 = $116)

An investor wants to make an initial purchase of $3,000 in a margin account. How much equity are they required to deposit? A. $1,000 B. $1,500 C. $2,000 D. $3,000

C. $2,000 Under FINRA rules, if a customer wants to purchase between $2,000 and $4,000 of stock in a margin account, they must deposit minimum initial equity of $2,000

Which of the following is equal to 9 basis points? A. 0.000009 B. 0.009 C. 0.0009 D. 0.0000009

C. 0.0009 Is is important to note that this answer is a percentage. One basis point is equal to .01% or 0.0001. There are 100 basis points in 1%. To convert basis points to a percentage. move the decimal two places to the left. To convert basis points to a decimal, move the decimal point four places to the left. So, 9 basis points becomes .09% or 0.0009

Martin opens his first Roth IRA on March 31, 2014, at age 56. What is the first date on which he can take a qualified distribution that is tax-free? A. 31-Mar-19 B. When he turns age 59 1/2 C. 1-Jan-19 D. 31-Mar-17

C. 1-Jan-19 Qualified distributions are available after meeting 2 criteria 1. Five years after the first contribution(which is deemed to occur on Jan 1 of that year) 2. After the account holder's 59 1/2 birthday. In this case, five years after the first contribution is January 1, 2019 (Jan 1 2014 is year 1 and Dec 31, 2018 is the end of year 5. In 2019 Martin is also older than 59 1/2

Material event notices must be submitted to the MSRB within A. 30 days of the event B. 4 business days of the event C. 10 business days of the event D. 24 hours of the event

C. 10 business days of the event Under SEC Rule 15c2-12, issuers must submit material event notices to the MSRB within 10 business days of the event. Examples of such events include rating changes, defaults, and unscheduled draws on credit enhancements.

Registered representatives are required to re-qualify by examination if their registrations are not sponsored by a member firm for a period of A. 6 months B. 1 month C. 2 years D. 1 year

C. 2 years An individual must take and pass the appropriate qualification exam if he or she has not been sponsored by a member firm for a period of two years or more

Juanita is a school teacher who defers her own money into a tax-sheltered annuity (TSA). She directs her own investment choices, among those offered by the annuity. What type of plan is it? A. 401(k) B. Defined benefit C. 403(b) D. SIMPLE

C. 403(b) 403(b)s are participant-directed ERISA plans sponsored by non-profit organizations and educational institutions. Like 401(k)s, they allow participants to make elective deferrals. When they are invested exclusively in annuities, they are called tax-sheltered annuities (TSAs)

ABC Corporate debenture is trading at $1,020 and pays a $40 semiannual coupon. What is the current yield of this bond? A. 3.9% B. 6.2% C. 7.8% D. 8%

C. 7.8% The current yield of a bond is found by dividing the annual interest by the bond's market price. In this case, we divide the annual interest, which is $80 ($40 semiannual coupon x2), by the market price of $1,020, to arrive at the current yield of 7.8%

For which of the following investors is a variable annuity most appropriate? A. A 55-year old that is planning to retire in 5 years, and can tolerate no loss of investment principal B. A 45-year old that is looking for an investment opportunity for retirement savings that offers growth potential, tax deferral, and complete liquidity C. A 60-year old that would like an investment that can provide income for life with the potential to keep pace with inflation D. A 35-year old that is looking for an investment option that provides tax-deferred growth

C. A 60-year old that would like an investment that can provide income for life with the potential to keep pace with inflation Variable annuities are designed for retirement savings. Of these choices, the 60-year old seeking retirement income that keeps pace with inflation seems to be well suited for this product. Investors that require full liquidity or are uncomfortable with potential loss of principal are not good candidates for variable annuities. Because of their lack of liquidity, annuities are not generally recommended for younger investors

A banker:s acceptance (BA) is a time draft drawn on A. A financial intermediary counterparty B. A life insurance company C. A deposit at a bank D. Corporate assets posted as collateral

C. A deposit at a bank A BA is a short-term negotiable debt instrument issued by a borrower and guaranteed by a commercial bank. Technically, it is a time draft drawn on a deposit at a bank

Most hedge funds are offered to investors through A. A secondary offering B. An IPO C. A private placement D. A shelf offering

C. A private placement Hedge funds are sold through private placements to accredited and institutional investors. They are exempt from the SEC registration process for public offerings

All of the following events are reportable to FINRA within 10 business days of occurrence EXCEPT A. A written customer complaint alleging forgery B. A representative's conviction of a securities industry misdemeanor C. A representative's involvement in an outside business activity D. Proceedings against the firm brought by a securities industry regulator alleging any rule violation

C. A representative's involvement in an outside business activity A representative's involvement in a private business activity is not an event that is required to be reported to FINRA promptly or within 10 business days of occurrence. Events requiring reporting within 10 days usually involve a violation or a serious allegation

A mutual fund share that charges 12b-1 fees is which of the following? A. B-share B. C-share C. A, B, or C-share D. A-share

C. A, B, or C-share 12b-1 shares are charged by mutual funds for marketing and distribution. These charge are in addition to front-end or back-end sales charges

Which of the following information must be included in the proxy materials that are filed with the SEC and distributed to shareholders? A. The most recent tax returns B. The slate of nominated C. All important facts upon which shareholders are asked to vote on D. The most recent financial statements

C. All important facts upon which shareholders are asked to vote on Proxy materials are required public filings that give shareholders the essential information they need to cast informed votes. They must be filed prior to any required shareholder votes

A broker-dealer is required to develop an Anti-Money Laundering Program that includes all of the following EXCEPT A. Updates to FINRA of any change in status to the firms AML officer within 30 days of the change B. Annual independent testing by a qualified party C. An annual year-end filing with FINRA that provides a detailed overview of any changes to the firms AML plan for the following year D. Ongoing training for all appropriate personnel

C. An annual year-end filing with FINRA that provides a detailed overview of any changes to the firms AML plan for the following year FIRNA requires that firms implement Anti-Money Laundering programs and controls. An Anti-Money Laundering compliance officer must be named, and any changes relating to that person must be updated with FINRA within 30 days of the change. All appropriate personnel within the firm must receive ongoing anti-money laundering training. A firm must have an independent testing of its controls annually if it holds customer accounts. Firms are not required to file a written report of proposed changes to their AML program with FINRA

Company A and Company B are multinational corporations whose typical business interests are in the import/export fields. The international transactions in which these businesses engage might be financed through the use of A. Negotiable CDs B. T-Bills C. BAs D. ADRs

C. BAs Internal transactions can be financed through the use of banker's acceptances (BA). These are typically checks drawn on a bank by an importer or exporter of goods and represents the bank's promise to pay the face amount of the note at maturity, which is usually within a three month time period. Banker's acceptances are frequent investments in money market funds

FINRA BrokerCheck includes which of the following information? A. Personal descriptions of each registered representative included in the BrokerCheck database B. Information about any representative who has been active in the past 15 years C. Bankruptcy proceedings within the last 10 years D. A broker-dealer's financial reports that are filed with FINRA on a quarterly basis

C. Bankruptcy proceedings within the last 10 years FINRA BrokerCheck includes bankruptcy information for the past 10 years and other licensing and registration information submitted to FINRA (Form BD and BDW for firms; Forms U4 and U5 for representatives) It does not provide details of financial information from quarterly FINRA filings or personal descriptions of representatives. It also includes information about any representative who has been active in the past 10 years, not 15 years.

An individual is currently contributing to her corporate retirement account. During this time she may A. Only make after-tax contribution to her corporate retirement account B. Only make after-tax contributions to her IRA C. Contribute to an IRA D. Not contribute to an IRA

C. Contribute to an IRA An individual is permitted to contribute to both a corporate retirement plan and an IRA at the same time

As the result of rising CPI, it would be anticipated that bond prices will A. Remain stable B. Increase C. Decrease D. Become volatile

C. Decrease Strong economic growth will tend to cause bond prices to fall

It would be inappropriate for an individual with a low net worth to A. Buy mutual funds B. Buy US Government bonds C. Engage in short selling D. Buy a broad mixture of stocks and bonds

C. Engage in short selling short selling can be a very risky investment strategy and is not typically suitable for individuals who have a low net worth

All of the following securities pay interest that is taxable at the federal level and exempt from state taxation EXCEPT A. Federal Farm Credit Bank bonds B. Treasury Bills C. Federal Home Loan Mortgage corporation notes D. TIPS

C. Federal Home Loan Mortgage corporation notes Please note that this is an except question. Issues of the federal government like Treasury notes, bonds, bills and TIPS pay interest that is taxable at the federal level but exempt at the state level. Federal Farm Credit and Federal Home Loan Bank securities follow this same taxation rule. Government National Mortgage Securities (Ginnie Maes), Federal National Mortgage Association securities (Fannie Maes) Federal Home Loan Mortgage Corporation securities (Freddie Mac) are taxable at all levels.

The separate account of an insurance company is managed to A. Protect purchasers of variable insurance products from investment risk B. Deliver guaranteed lifetime income to purchasers of either fixed or variable annuities C. Help provide inflation protection for purchasers of both life insurance and annuity products D. Offer principal protection to purchasers of variable products

C. Help provide inflation protection for purchasers of both life insurance and annuity products The separate account of an insurance company separates purchase payment for variable life and annuity products form purchase payments for fixed products (general account products). The separate account is designed to offer growth to keep pace with inflation for purchasers of variable products. In exchange for growth potential purchasers take on investment risk.

If a municipal finance professional makes a $350 contribution to the campaign of an issuer official who she is entitled to vote for, A. Her firm cannot do any negotiated underwriting with any issuer for the next two years B. Her firm may not do any municipal securities work with that issuer for the next two years C. Her firm cannot do any negotiated underwriting with that issuer for the next two years D. A two-year ban on competitive underwriting for that issuer will be triggered

C. Her firm cannot do any negotiated underwriting with that issuer for the next two years The MSRB political contribution rule prevent municipal securities broker-dealers from engaging in any negotiated underwriting with an issuer, where the firm or a municipal finance professional of that firm, makes a contribution of more than $250 to an official of that issuer. If the professional is not eligible to vote for the issuer official, they may not contribute any cash to the officials campaign

Which of the following investment products would not be appropriate for an individual who says her current investment objective is growth? A. Small-cap stock B. Equity mutual fund C. High-grade municipal bond D. Exchange-traded fund

C. High-grade municipal bond Growth-oriented investors would be interested in equities and equity mutual funds. Fixed-Income products would not be a suitable investment for this investment objective

Which of the following contracts are subject to automatic exercise at expiration if the market price of XYZ is $45.25? I. Long XYZ 42 call II. Long XYZ 46 put III. Long XYZ 48 call IV. Long XYZ 44 put A. I & IV B. II & III C. I & II D. III & IV

C. I & II The Options Clearing Corporation has provisions for the automatic exercise of certain in-the-money options at expiration. Generally, the OCC will automatically exercise any expiring equity call or put in a customer account that is $0.01 or more in-the-money, and an index option that is $0.01 or more-the-money. Calls are in the money when the market price is higher than the exercise price; puts are in the money when the market price is lower than the exercise price.

Which two of the following options can be combined to create a Covered Call? I. Long Stock II. Long Call III. Long Put IV. Short Call V. Short Stock A. I & III B. II & V C. I & IV D. IV & V

C. I & IV A covered call is made up of a long stock position and a short call position. Covered call writing is a strategy used by investors with long stock positions to generate additional income. By writing calls, premiums are received and if the market price of the stock does not surpass the exercise price, the call will not be exercised

Ralph participates in a type of plan in which all contribution are made by the employer. Each year, the employer contributes a percentage of salary for all eligible employees and the percentage varies year to year. What type of plan is it? A. Money purchase B. 401(k) C. Profit-sharing D. SIMPLE

C. Profit-sharing Profit-sharing plans give employees the flexibility to vary the level of annual contributions based on the company's profits

Which of the following is Bearish? A. Protective Put B. Covered Call C. Protective Call

C. Protective Call

Which of the following parties are classified as insiders under the Securities Exchange Act of 1933? I. A shareholder who owns 7 percent of outstanding stock of a corporation and his spouse who owns 4 percent II. The vice president of a firm III. A firm's director of technology A. I, II, and III B. I and II only C. I only D. III only

C. I only The Securities Exchange Act of 1934 defines insiders as corporate directors, officers, or stockholders owning more than 10% of a firm's shares. The positions of spouses are aggregated in determining the ownership percentage of a shareholder. For some higher level ownership percentage of a shareholder. For come higher level employees like a VP or director of technology, they may be a corporate office for purposes of signing contracts or speaking on behalf of the company, but they are not considered "corporate insiders" under the narrower definition in the '34 Act.

Which of the following is most bearish? I. Long Call II. Long Put III. Short Call IV. Short Put A. I & II B. I & IV C. II & III D. II & IV

C. II & III

The typical fee compensation paid to a private equity manager is I. 1% of the invested capital II. 2% of invested capital III. 10% of the fund's net profits IV. 20% of the fund's net profits A. I and III B. I and IV C. II and IV D. II and III

C. II and IV A private equity fund manager is typically compensated with an annual management fee of 2% of the invested capital and 20% of the fund's net profits. This compensation is similar to that paid to hedge fund managers

If the Federal Reserve determines that a tight money policy is necessary, all of the following are likely EXCEPT: A. The exchange value of the dollar will rise B. Stock prices will fall C. Interest rates will fall D. Bond prices will fall

C. Interest rates will fall A tight money policy exists when the Federal Reserve makes credit less available to slow down the economy. This means it will tighten the money supply through raising interest rates. As interest rates go up, bond prices fall and generally the stock market will also fall. The dollar's value against foreign currency will generally move in the same direction as interest rates.

Characteristics of commercial paper include all of the following EXCEPT A. Unsecured obligation of the issuer B. Usually matures within 270 days or less C. Issued only by banks and other financial institutions D. Highly liquid

C. Issued only by banks and other financial institutions Commercial paper is issued by corporations for short term financial needs. It is unsecured and usually matures in 270 days or less. It is usually sold in minimum denominations of $100,000 and is highly liquid

A free writing prospectus is used by a(n) A. Managing underwriter to invite broker-dealers into a syndicate B. Market maker to establish a quotation for a security C. Issuer to provide additional information to investors beyond what appears in the registration statement D. Issuer or underwriter to solicit orders from the public to purchase shares in an upcoming offering

C. Issuer to provide additional information to investors beyond what appears in the registration statement A free writing prospectus is used by an issuer to provide additional, updated information to investors beyond what the registration or prospectus contains, with the goal being to provide investor with pertinent updates without having to amend their registration statement or prospectus

What is the best way to describe an investor's profit potential when writing a call option? A. It is limited to the premium paid B. It is significant if the underlying stock declines C. It is limited to the premium received D. It is unlimited if the underlying stock increases

C. It is limited to the premium received The maximum gain for the writer of an option is the premium received

Joe is a registered rep currently employed by Windy City Broker-Dealer, a Chicago based firm. Joe has quite the sweettooth. To satisy his sugary cravings, he decides to open Joe and Joesettes Cupcakery, a small bakery in a hip part of town. Joes business partner, Josette, also happens to be a client of the firm and recently borrowed $5,000 from Joe. Which of the following is true regarding this action? A. Opening the Cupcakery requires permission from the firm, since it is an outside business activity B. Open the Cupcakery does not require permission, but lending money to Josette is prohibited C. Lending money to Josette is permitted provided that the firm has written procedures permitting borrowing and lending with clients D. Joe can open the Cupcakery and lend money to Josette provided that the firm is notified of both activities

C. Lending money to Josette is permitted provided that the firm has written procedures permitting borrowing and lending with clients Any outside business activity requires that the firm be notified of the full details of the activity. FINRA Rule 3240 permits a registered rep to borrow money from or lend money to a client provided that the firm has written supervisory procedures permitting such an activity. If the client happens to be a bank or family member, the firm may permit the activity without permission. If there is an outside personal or business relationship, the registered rep must receive permission from the from the firm to borrow or lend to a client

A customer sells short 100 shares of XYZ stock for 72 and buys one XYZ 75 call for 1.50. The stock price rises to 77 and the option is exercised. The profit or loss to the investor is A. Profit of $450 B. Loss of $650 C. Loss of $450 D. Profit of $650

C. Loss of $450 The stock is sold short for $7,200. To protect the position the investor buys a call for $150. The call is exercised when the market price of the stock rises, so the investor buys the stock to cover the short position for $7,500. The customer received $7,200 from the short sale, but paid a total of $7,650 (premium + stock purchase price) for a loss of $450

All of the following enforce municipal securities industry rules EXCEPT A. Office of the Comptroller of the Currency B. SEC C. MSRB D. FINRA

C. MSRB Enforcement and inspection for compliance with MSRB rules is done by the SEC and FINRA for brokerage firms and by the FRB, FDIC, and the Office of the Comptroller of the Currency for dealer banks

A business structure that offers limited liability and pass through benefits but is publicly traded is a: A. Limited Liability Corporation B. Subchapter S Corporation C. Master Limited Partnership D. C-Corp

C. Master Limited Partnership A master limited partnership (MLP) is a pass-through taxation entity, but has ownership divided into shares that can be publicly traded

What happens to a buy stop order before the ex-date of a stock split? A. It must be adjusted in both price and number of shares B. It must be adjusted in number of shares only, not price C. Nothing D. It must be adjusted in price only, not number

C. Nothing Buy stop orders are entered above the current market price, and are not adjusted in the event of a forward stock split (or cash dividend). If the split had been a reverse split then all open orders are cancelled

Under MSRB rules, all of the following would be considered advertisements EXCEPT A. Market letters B. Summaries of official statements C. Official statements D. Offering circulars

C. Official statements According to the MSRB, official statements are not considered advertisements. Official statements are provided to purchasers of a new issue municipal bond. Their creation is optional, but if they are created, every investor must receive one

For every security on the NYSE, there is (are) A. Several institutional traders B. A limit to how many shares of that security may trade on a given business day C. One designated market maker D. Multiple Market Makers

C. One designated market maker Every NYSE-listed Security has one designated market maker assigned to it. In the Nasdaq marketplace, there may be several market makers trading that particular security

Which of the following parties is not permitted to purchase an IPO? A. Bank holding company manager B. Telemarketer for a bank website C. Portfolio manager of a mutual fund for his personal account D. Employee of an insurance company

C. Portfolio manager of a mutual fund for his personal account There are several parties who are not permitted to purchase an IPO, including the portfolio manager of a mutual fund for their personal account. All entities who cannot purchase IPOs are broadly characterized as restricted persons

Personal incomes are an example of an economic indicator that is A. Acyclic B. Defensive C. Procyclic D. Countercyclic

C. Procyclic Procyclic indicators move in the same direction as the general economy; they increase when the economy is thriving and decrease when it is performing poorly. Gross Domestic Product (GDP) is another example of procyclic indicator

At what age must Required Minimum Distributions (RMDs) from a Roth IRA begin during an account owner's lifetime? A. 75 B. 59 1/2 C. RMDs are never required during the account owner's lifetime D. 72

C. RMDs are never required during the account owner's lifetime RMDs are never required from Roths during the account owner's lifetime. This is different than Traditional IRAs, which require RMDs to begin in the year after the account owner turns 72.

Which of the following funds subject investors to a broad range of equity securities with the lowest management fees? A. A global stock mutual fund B. A DJIA Index ETF C. S&P 500 Index ETF D. A diversified domestic closed-end fund

C. S&P 500 Index ETF Index funds traditionally carry the lowest management fees since there is little active fund management. The S&P 500 offers a broader range of companies than the DJIA Index which includes 30 industrial stocks

To partially protect a long stock position and increase income to her portfolio, an invest could do which of the following? A. Buy Calls B. Buy Puts C. Sell Calls D. Sell Puts

C. Sells Calls Additional portfolio income can be generated by writing calls against a stock position that is currently held. The premium from the calls delivers the additional income. (Covered Call)

In which of the following positions does the investor face unlimited risk? A. Long Call B. Long Put C. Short Call D. Short Put

C. Short Call when an investor is short a call with no other positions, the investor is obligated to buy stock if assigned. There is no limit on the price to which the stock can rise, so the investor's risk is theoretically unlimited

An investor with a high-risk tolerance would like to add a growth fund to his portfolio. Of the following, which type of fund may be most appropriate? A. Hybrid fund B. Junk bond fund C. Small cap stock fund D. Commodity fund

C. Small cap stock fund A stock fund is most appropriate for growth objective. Because the investor is not risk averse, a small cap fund will offer the most growth potential of the choices offered

Term, or dollar bonds, mature at a specified date in the future, commonly known as the maturity date. Term bonds can also be repaid prior to maturity if: A. If the company charter has a provision for early repayment of the bonds B. The CEO of the company authorizes early retirement of the debt C. The bonds are called by the issuer D. The company's stockholder's approve of the action

C. The bonds are called by the issuer Bonds can be repaid prior to their stated maturity date if they are called by the issuer

An investor writes an ABC January 50 put at 3. At expiration ABC stock is trading for 47. Which of the following statements is TRUE? A. The contract is out of the money B. The contract has an intrinsic value of $600 C. The contract has intrinsic value of $300 D. The contract has no intrinsic value

C. The contract has intrinsic value of $300

A journalist who worked for a business magazine learns about the confidential takeover of a company he is writing about while performing his work duties. As a result, he buys a large amount of stock in the company, but then learns that the takeover will not take place and subsequently sells the stock position and ultimately breaks even on the transactions. In this situation A. A violation of insider trading rules may have occured because the journalist is considered an insider of the company due to his access to company confidential information B. A violation of insider trading did not occur because the journalist did not profit on the transaction C. The journalist may have committed a violation under the misappropriate theory D. A violation of insider trading rules did not occur because the journalist is not considered an insider of the target company

C. The journalist may have committed a violation under the misappropriate theory In this case the journalist is not an insider of the company targeted for takeover. However, under the misappropriation theory, employees have fiduciary responsibility toward, and cannot misuse, confidential information about other companies entrusted to them by their employer. In this case the journalist clearly violated the misappropriation theory

The terms and conditions of the public offering and the agreement between the issuer and the underwriting manager are formalized in a contract known as the: A. Syndicate Letter B. Selected Dealer Agreement C. Underwriting Agreement D. Agreement Among Underwriters

C. Underwriting Agreement The underwriting agreement defines the terms between the issuer and the underwriting manager in public offering

Which of the following investment company types does not have an investment advisor? A. REIT B. Open end company C. Unit Investment Trust D. Closed end company

C. Unit Investment Trust Of the three basic types of investment companies, unit investment trusts are without an investment advisor. They are not actively managed. REITs are not defined as an investment company under the investment company act of 940 because they invest in real estate not securities

A brokerage firms Customer Identification Procedures require all of the following EXCEPT A. Obtaining customer identification documentation from each customer prior to account opening B. Determining whether the customers name appears on the OFAC list of known or suspected terrorists or terrorist organizations C. Verification of customer identity prior to account opening D. Maintenance of records of the documents used to verify identity

C. Verification of customer identity prior to account opening A firms Customer Identification Procedures require that the customers identity is verified either before or within a reasonable time of account opening

James retired several years ago, but has continued to work part-time in his family's store, earning a small amount. He receives Social Security. Can he contribute to an IRA? A. No, because he is receiving Social Security B. Yes, but only until he turns age 65 C. Yes, he is allowed D. Only if he earns above a dollar limit

C. Yes, he is allowed Individuals who work and earn compensation can contribute to an IRA whether or not they receive Social Security

A sell stop order is entered A. above the current market price B. either above or below the current market price C. below the current market price D. at the current market price

C. below the current market price A sell stop order is entered below the current market price to protect a loss in a long position, or is entered in anticipation of a stock breaking below a support level and decreasing significantly in price

A proven strategy for mitigating the systematic risk in a portfolio of US stocks is to A. emphasize dividend-paying stocks B. reduce stock holdings gradually during market downturns C. buy put options on a broad-based stock market index D. diversify among many different stock issues

C. buy put options on a broad-based stock market index Buying put options on a broad-based index can generate profits when all stocks are falling that helps to mitigate market risk

A corporate bond that has a sinking fund provision A. has more default risk than other corporate bond issues without this provision B. is usually retired in whole at the discretion of the trustee C. can be offered at a lower interest rate than other bonds D. is exempt from the Trust Indenture Act of 1939

C. can be offered at a lower interest rate than other bonds Bonds that are retired through a sinking fund have less default risk, because the issuer is making payments in advance to a trustee to buy bonds in the open market. This reduction of principal risk allows the issues to offer these bonds at a lower interest rate.

Asset Backed Securities cold be backed by all of the following EXCEPT A. credit card receivables B. student loans C. corporate equipment D. auto loans

C. corporate equipment ABS consist of financial assets that are packaged together and sold to investors as bonds. Corporate equipment is not a financial asset and, therefore, would never be securitized into an ABS

Investors who diversify their portfolios with companies in different industries are typically able to avoid A. marketability risk B. systemic risk C. non-systemic risk D. market risk

C. non-systemic risk Non-systemic risk is a company or industry specific risk that is inherent in each investment. No systemic risk can be reduced through appropriate diversification

A final prospectus must always include A. underwriting discounts or commissions available to syndicate members B. discounts available to member firms and dealers C. the offering price D. the Statement of Additional information

C. the offering price A final prospectus includes all relevant information regarding the offering including the final offering price. It does not contain information about the underwriting concessions or discounts available to member firms through selling agreements. The Statement of Additional information, often referred to as Part B of the registration statement, is not included in the prospectus, though it is available to investors who request it

A husband and wife, who file a joint return, have compensation of $200,000. Neither of them is covered by a retirement plan at work. What part of their Traditional IRA contribution are deductible? A. None B. Half C. 100% of contributions made to one IRA D. 100% of contributions made to both IRAs

D. 100% of contributions made to both IRAs There are two tests of the ability to make deductible contributions to a Traditional IRA. One is based on whether the IRA owner, or the owner's spouse, is covered by a retirement plan at work. In this case the contribution would not be tax deductible. The other is based on their income. If neither spouse is covered at work, then both may make deductible contributions

An investor sells 10 XYZ June 30 puts at 3.5. The breakeven point is A. 350 B. 33.5 C. 30 D. 26.5

D. 26.5 The breakeven point for the seller of a put option is the strike price of the option minus the premium received for the option

When in the process of hiring a registered representative, an employing firm must check the employment history for the previous A. 10 years B. 2 years C. 5 years D. 3 years

D. 3 years Firms are required to check the employment history of persons they will sponsor for registration for the previous three years even though the rep will disclose the last 10 years

A broker-dealer is required to file notice of the resignation of a representative to FINRA within A. 20 days B. 15 days C. 60 days D. 30 days

D. 30 days Broker-dealers are required to file a Form U-5 notice of representative termination with FINRA within 30 days. If the rep is terminated with cause, the termination must be reported within 10 days

Which of the bonds below would subject an investor to the greatest amount of call risk? A. 4% Bond with a call premium of 102 B. 6% Bond with a call premium of 102 C. 4% Bond with no call premium D. 6% Bond with no call premium

D. 6% Bond with no call premium An issuer is most likely to call bonds with a higher coupon when interest rates are falling. AN issuer also prefers to call bonds that do not require payment of a call premium.

An institutional portfolio includes commercial paper, variable-rate demand obligations, and in-state AAA municipal bonds. The investment objective of the portfolio could include all of the following EXCEPT A. Tax-exempt income B. Liquidity C. Safety of principal D. Capital appreciation

D. Capital appreciation The securities in this portfolio offer safety of principal, tax exempt income, and liquidity. Growth, capital appreciation, and speculation are objectives that are not achieved with this portfolio

A T Bond pays interest on the 15th of April & October. A trade occurs on Friday January 8. How many days of accrued interest does the buyer owe the seller? A. 85 B. 86 C. 89 D. 88

D. 88 We must use actual days when determining accrued interest on Treasury notes and bonds. Treasuries use next business day settlement (not two business days as in the case with corporates and municipals). The counting convention is to begin from (and including) the last coupon payment date and up to but not include the settlement date of the trade (The buyer of the bond begins earning his own interest on the settlement date) In this example, we have 17 days for October, 30 days for November, 31 days for December, and 10 days for January. Remember, with the trade occurring on a Friday and the settlement date being Monday, we must include Saturday and Sunday. The total is 88 days.

An investor is interested in making annual payments to purchase an annuity that will provide income in 10 years. This investor should purchase a(n) A. A single premium deferred annuity B. Immediate annuity C. A periodic payment future annuity D. A periodic payment deferred annuity

D. A periodic payment deferred annuity In annuity terminology, multiple payments are called periodic payments. Deferred annuities pay income in the future.

The general eligibility standard for ERISA plans includes which components? A. Seniority and work service B. Position and salary level C. Age and salary D. Age and work service

D. Age and work service In general, ERISA requires plan eligibility for any employee who is at least 21 years old with one year of work service with the employer

Which provisions of the Securities Act of 1933 prohibit deceit and misrepresentations in the offering or sale of securities? A. Continuous reporting provisions B. Public registration provisions C. Blue sky provisions D. Antifraud provisions

D. Antifraud provisions The antifraud provisions of the Securities Act of 1933 prohibit deceit, misrepresentations and other fraud in securities offers and sales

XYZ is trading at 60. Which of these options contracts is trading for time value only? A. June 65 Put B. May 55 Call C. July 50 Call D. August 55 Put

D. August 55 Put An option will trade for its time value only when it is out of the money. In this example, only the August 55 put is out of the money. The other options positions are in the money

What is the correct sequence of events, from the first to last, in the registration timeline? A. Registration statement filed, effective date, road show, bake-off B. Road show, bake-off, effective date, registration statement filed C. Road show, registration statement filed, bake-off, effective date D. Bake-off, registration statement filed, road show, effective date

D. Bake-off, registration statement filed, road show, effective date The timeline begins with pre-registration period events such as a bake-off to choose the lead underwriter. It ends with the effective date, which is when the SEC clears the securities for public sale

Which of the following individuals is not eligible to open a Roth IRA and make a contribution to it? A. Paul who is age 12 and makes money mowing lawns B. Alice, age 75 who is married to a person who works C. Doris, who is age 55 and is a self-employed artist D. Bradley, who is age 45 and earns $500,000 a year as a banker

D. Bradley, who is age 45 and earns $500,000 a year as a banker Anyone who works and earns compensation (or is married to a worker with compensation) can make a Roth contribution, except a person who has compensation above a threshold.

ForeignCo, with offices in 15 countries, including the US, engages Jim Securities, Inc. as an advisor to make corporate acquisitions for cash and securities. Shortly after signing an engagement letter, ForeignCo's CFO gives Robert, a banker on the deal team, a mandate to screen for potential acquisition targets with significant business in Cuba, Venezuela and North Korea. At this point, Robert should A. File an SAR with FinCEN and notify only his employer that he is doing so B. File an SAR with FinCEN and notify his employer and ForeignCo that he is doing so C. File an SAR with FinCEN without notifying whether his employer or ForeignCo D. Bring the issue to the attention of his manager, who will make a determination of whether an SAR should be filed

D. Bring the issue to the attention of his manager, who will make a determination of whether an SAR should be filed Although any suspicious activity, especially with countries hostile to the US requires the submission of a Suspicious Activity Report, it is ultimately the decision of a principal or the firm:s designated AML. Compliance person to file such a report. In this question the best answer is that Robert should bring the attention to his superiors if an SAR is ultimately filed with FinCEN, the client is not notified of this event.

Which of the following securities does not trade on the OTCBB? A. Warrants B. Foreign equity issues C. DPPs D. Debentures

D. Debentures OTCBB securities include national, regional, and foreign equity issues, warrants, units, American Depositary Receipts (ADRs), and Direct Participation Programs (DPPs). Debentures are debt securities; only equity securities are included in the OTCBB

Which of the following is not an ERISA Qualified plan? A. Corporate Pension Plan B. Keogh Plan C. 401(k) Plan D. Deferred compensation plan

D. Deferred compensation plan A deferred compensation plan is a type of non-qualified plan. In these types of plans, only after tax funds may be contributed

Karen participates in an employer retirement plan that is 100% funded by the employer and promises to pay her a steady retirement income based on her salary and years of work service. What type of plan is it? A. Defined contribution B. 401(k) C. Money purchase D. Defined benefit

D. Defined benefit Defined benefit plans are 100% funded by the employer and promise to pay participants an income benefit at retirement age. The benefit may be based on a formula that takes into account salary, age and years of work service

If a margin account customer has failed to meet the Reg T settlement date because of special circumstances, a broker-dealer may request an extension from A. the Federal Reserve Board B. the SEC C. Securities Investor Protection Corporation D. FINRA

D. FINRA If a Reg T call has not been met due to special circumstances, a broker-dealer can request an extension from its SRO, which is typically FINRA

Advantages of book entry form of ownership for US government securities include all of the following EXCEPT A. Increased protection for investors from loss and theft of securities B. Ease of transfer between parties when securities are bought and sold C. Reduced printing costs for the US government D. Faster regular way settlement terms than apply to government securities issued in other forms

D. Faster regular way settlement terms than apply to government securities issued in other forms All Treasury securities are issued in book entry form, which reduces printing costs for the US government and supplies increased protection and ease of the transfer for investors and the securities firms they do business with

What form does an RIA file to register with the SEC? A. Form 15-F B. Form ATS C. Form SF-3 D. Form ADV

D. Form ADV Form ADV is the Uniform Application for Investment Adviser Regulation. After it is filed, the SEC has 45 days to review the information and determine whether it should be approved

A municipality is planning to raise money to finance a large government building complex. It hires a financial advisor to provide assistance in planning the new issue. If the advisor believes that interest rates may soon trend higher, the advisor would most likely recommend which one of the following A. Revenue bonds with medium-term maturities B. TANs C. BANs D. GO bonds with long-term maturities

D. GO bonds with long-term maturities Because interest rates are on the rise, the financial advisor will most likely recommend locking in the lowest possible rate for the long-term. Therefore, it is likely to recommend long-term bonds to finance the project. GO bonds are used to finance public projects like government buildings

AN 80-year-old individual would be least likely to purchase a A. Money Market Fund B. Treasury Bond C. Bank CD D. Hedge Fund

D. Hedge Fund An individual who is retired, or in the later stages of life, would not be likely to make an investment that could result in the complete loss of their capital or one that would require a long-term investment horizon

Which two of the following describe the process of assignment of exercise notice correctly? I. Exercise notices are assigned to one or more clearing member with short positions II. Exercise notices are assigned to one or more clearing members with long positions III. The OCC assigns exercise notice to clearing members on a FIFO basis IV. The OCC assigns exercise notice to clearing members based on its established procedures A. II and III B. II and iV C. I and III D. I and IV

D. I and IV Exercise notices are assigned against short options positions, which are then obligated to deliver. The OCC has written procedures which detail the process for assigning the exercise of options contracts. The clearing member firms then assign exercise notice to customer either randomly or on a first-in, first-out (FIFO) basis

If a representative fails to complete Regulatory Element when required I. Registration status is inactive II. Registration is terminated III. All business activities must cease IV. All business activities that required registration must cease A. II and IV B. I and III C. II and III D. I and IV

D. I and IV When a representative fails to complete Regulatory Element when required, his registration status is inactive, and the representative cannot perform, or be compensated for, activities that require registration

Which of the following types of communications with the public are permitted during the cooling-off period? I. Red Herring II. Road Show Presentation III. Tombstone Ad A. I and II only B. I and III only C. II and III only D. I, II, III

D. I, II, III During the cooling-off period, communications generally are limited to the preliminary prospectus (red herring), tombstone ad, and a conforming road show presentation

A customer has purchase $5,000 of securities in a cash account and does not pay for them on time. Which TWO of the following actions is the broker-dealer permitted to take in this situation? I. Waive the call because of the amount II. Request an extension if special circumstances apply III. Allow additional sales and purchase transaction to take place in the account IV. Sell the securities out and freeze the account for 90 days A. I and IV B. II and III C. I and III D. II and IV

D. II and IV If a customer has purchased securities in a cash account and does not pay for them on time, the broker-dealer may either ask the SRO for an extension for additional time on behalf of the customer, or sell the securities out and freeze the account for 90 days

A broker-dealer may compensate an outside party for a business referral in the form of finder:s fee, A. Under no circumstance B. If the payment is recorded on the broker-dealers books C. If a written agreement is in place between the outside party and the broker-dealer D. If the outside part is properly registered

D. If the outside part is properly registered Securities industry regulations prohibit the payment of any compensation to unregistered persons

All of the following generally accompany a slowing economy EXCEPT: A. An increase in the money supply B. An increase in purchases of securities by the Fed's Open Markets Committee C. Stabilizing or falling inflation D. Increased demand for credit

D. Increased demand for credit Demand for credit increases when the economy is strengthening, or in an expansionary cycle. The Fed puts more money into the money supply when it purchases securities. It does this to help increase the supply of money to reduce interest rates, and stimulate demand for credit. Inflation is usually stable or falling during period of economic decline

Which of the following mutual funds is most likely a "no-load" fund? A. High-yield B. Sector fund C. Target date fund D. Index fund

D. Index fund Most index funds operate as "no-load" funds

Which of the following is a significant risk of a raw land limited partnership program that must be disclosed by a registered representative? A. Limited appreciation potential B. All of these are risks of raw land partnerships that must be disclosed C. Significant tax losses that are only useful to investors that have other passive income D. Lack of liquidity

D. Lack of liquidity a raw land partnership has high appreciation potential. It does not pass through losses because there are no expenses associated with development of the land of depreciation. The lack of liquidity is a risk that must be disclosed

The risk that stock cannot be sold easily or promptly is known as A. Reinvestment rate risk B. Financial risk C. Systemic risk D. Marketability

D. Marketability The risk that a security cannot be easily converted to cash is known as marketability

A security that can be freely transferred, assigned or delivered to another entity is called A. Viable B. Fungible C. Marketable D. Negotiable

D. Negotiable A negotiable security is one that can be freely transferred, assigned or delivered to another entity. Listed equities are negotiable when they are traded or assigned by an authorized owner, unless they are encumbered as debt or collateral

What part of a distribution from a Traditional IRA is not taxable as ordinary income? A. Capital gains B. The first $10,000 C. Interest and dividends D. Non-deductible contributions

D. Non-deductible contributions Tax has already been paid on non-deductible contributions. They are distributed tax-free, assuming that the non-deductible contributions have been properly reported to the IRS in the year they were made

TANS, RANS, and BANS are issued by municipalities for A. Pass-through financing B. Private purpose non-essential services funding C. Federally subsidized financing D. Short term financing

D. Short term financing TANS, RANS, and BANS are short term municipal securities that are issued in anticipation of longer term funding from taxes or other forms of municipal revenue. They are not federally subsidized or specifically for funding private purpose facilities

The stock of a company with no earnings history, but high potential for appreciation is most likely classified as a A. Income stock B. Mid-cap stock C. Cyclical stock D. Speculative stock

D. Speculative stock Speculative stocks generally have no earnings history or widely varying earnings, but they have high potential for appreciation because of their future promise

A tax deduction for contributions to a 529 plan may be available at the A. Federal and state level B. Neither the federal nor state level C. Federal level only D. State level only

D. State level only Many states offer state income tax deductions to their own residents for contributions to 529 savings plans. The limits on deductions are set by each state and vary from state to state

A broker-dealer is registered with FINRA. This means that A. It is safe to do business with the firm B. The services of the firm are endorsed by FINRA C. They are authorized by the SEC to engage in a securities business D. The firm is authorized to engage in the securities business

D. The firm is authorized to engage in the securities business FINRA registration confers an authority to engage in a securities business. It does not confer an endorsement or any other plug for the firm

When a registered rep shifts a customer from one mutual fund to a different fund with a similar investment objective A. they are helping the client to achieve stronger investment returns B. they are performing normal account maintenance, which typically involves the reallocation of cash from one mutual fund to another C. They are conducting an illegal activity, which may result in a suspension from the securities industry D. They are engaged in "switching" a sales practice violation

D. They are engaged in "switching" a sales practice violation "Switching" occurs when a registered rep moves a customer from one mutual fund to another fund with a similar, or identical, investment objective. There is no benefit to the customer when this occurs, and is usually harmful

An investor sells a call option for a premium of $3 and a strike price of $85. If the market value of the underlying stock is $83, the option has an intrinsic value of A. $1.00 B. minus $5 C. minus $2 D. zero

D. zero Intrinsic value is the amount an option is in-the-money, which is the current market values, strike price for calls and strike price current market value for puts. I f an option is out-of-the-money, as in this example then intrinsic value is zero

A long standing client of your firm has just purchased $500,000 of Town X new general obligation bonds. In responding to this clients concerns regarding the potential loss of these funds, you might point that A. Your firm maintains SIPC insurance which will reimburse the client should the investment under perform B. They can enter into a repurchase agreement with the MSRB in the unlikely event of the default of the issuer C. All investments involve risk, but municipal bonds are generally safer than most other types of bonds so the client should not worry D. While you expect the investment to do well you cannot promise anything

D. While you expect the investment to do well you cannot promise anything Municipal securities broker-dealers and their associated persons may never guarantee a customer against a loss or tell the client that insurance is available to safeguard investors against investment losses

The Marshalls are a retired couple who trusted a broker to invest their IRAs. The broker then manipulated prices of trades in their accounts to line his own pockets. This happened two years ago, but the Marshalls only just learned about big losses in their accounts., Are they within the statute of limitations for bringing suit against the broker? A. Yes, because there is no statute of limitations on fraud B. No, the fraud happened too long ago C. No, they should have learned about it sooner D. Yes, because they are taking timely action

D. Yes, because they are taking timely action Investors who are injured by price manipulation can sue for damages and costs. The suit must be brought within three years of the manipulation and within one year of the discovery. This is the statute of limitation

An investor wishes to establish a college education fund for his son who is 3 years old. He is interested in ensuring that $100,000 will be available when the child reaches age 18. An instrument that may be well suited to meeting these objectives is a: A. Treasury Bond B. AAA Corporate Bond C. Blue Chip Common Stock D. Zero Coupon Bond

D. Zero Coupon Bond Of these choices a zero coupon bond can assure that a fixed amount of money is available at a particular time. Zero coupon bonds are purchased at a deep discount and mature to their face amount. They are well suited for planning for long-range goal, such as a child's education. Though a treasury bond is also a safe investment, the investor does not need the semi-annual coupon that it offers

An underwriting firm takes on risk in all of the following underwriting types EXCEPT a(n) A. standby B. competitive bid C. firm commitment D. all-or-none

D. all-or-none In an all-or-none underwriting, the underwriters are only responsible for the distribution if it is fully subscribed. If it isn't fully sold, the offering is canceled

A firm:s anti-money laundering program must be independently tested for compliance A. every three years B. quarterly C. semi-annually D. annually

D. annually FINRA rules require that a firm:s anti-money laundering compliance program is tested annually by a designated independent person

When a stock index call option is exercised the buyer receives A. a basket of stocks chosen by the buyer B. the stocks that make up the index C. either cash or stocks to be decided by the writer D. cash equal to the intrinsic value of the option

D. cash equal to the intrinsic value of the option When stock index options are exercised the buyer receives cash equal to the difference between the strike price and the current market value of the index (this difference is the intrinsic value) multiplied by the $100 multiplier

ABC Co. is seeking to have its securities listed on the OTCBB. In order to accomplish this, ABC must A. have generated a profit in at least two of its prior three fiscal years B. have a minimum number of shares in the public float C. secure an affirmative vote from at least a majority of its shareholders D. file current financial reports with the SEC

D. file current financial reports with the SEC Companies that wish to list their securities on the OTCBB must file current financial reports with the SEC. There is no requirement that the company be profitable or have a certain number of shares outstanding

Under the Securities Exchange Act of 1934, the Securities and Exchange Commission does all of the following EXCEPT A. regulate the solicitation of proxies B. provide rules for the segregation of securities owned by clients C. regulate the activities of members of national securities exchanges D. regulate the extension of credit by brokers-dealers

D. regulate the extension of credit by brokers-dealers The Securities Act of 1934 gives the Federal Reserve Board the right to regulate the extension of credit by brokers dealers (buying on margin)

Statutory disqualification can result in all of the following cases EXCEPT A. a firm imposed by FINRA on the disqualified representative or firm B. a disqualification from continued activities in the business of securities for a firm or rep already in business C. a disqualification of registration for a firm or representative D. up to ten years imprisonment

D. up to ten years imprisonment Statutory disqualification can impact a firm or representative either before they are registered or after they have begun to conduct business activities. The firm must discontinue the activities of someone in this status, unless they file an Eligibility Proceeding with FINRA to request continued sponsorship of the representative's activities. FINRA does not have criminal authority, though it could refer a case to the attorney general for criminal proceedings

A summary of the business continuity plan of a broker-dealer must be provided to each customer A. no later than with the delivery of the confirmation of the first trade B. within 24 hours of the opening of the account C. before the first trade is executed D. when the account is initially opened

D. when the account is initially opened This information must be provided to customers at the time the account is opened, and must be posted on the firm's website. Customers can always request a copy of this information to be mailed to them as well

A 10-year municipal bond, callable in 5 years is trading aat 97 7/8. Rank the following yields from lowest to highest I. Current Yield II. Nominal Yield III. Yield to Call IV. Yield to Maturity

II, I, IV, III when a bond is trading at a discount, nominal yield is lowest, then current yield, YTM and finally YTC

The short-term effect of the Federal Reserve's Open Market Committee is to A. check inflation B. stop a recession C. make credit more or less available D. do all of the above

The Fed buys and sells securities in the marketplace to increase or decrease the money supply, making credit more or less available. When the Fed buys securities it is putting money into circulation, increasing the money supply and making credit more available. When the Fed sells securities it is taking money out of circulation, decreasing the money supply and making credit less available. Long term, these actions may check inflation or stop a recession

Which of the following statements regarding equity REITs is TRUE? a. Dividends from equity REITs are typically paid on a monthly basis b. Dividends paid by equity REITs generally consist of both rental income and capital appreciation c. There are very few available to investors today d. They offer tax sheltering opportunities because they pass through losses to investors

b. Dividends paid by equity REITs generally consist of both rental income and capital appreciation Dividends paid by equity REITs consist of both rental income and capital appreciation from the properties that are owned. Dividends are typically paid annually from equity REITs. Equity REITs are the most common type of REIT today; they do not provide a tax shelter as they do not pass through losses to investors

a "breakpoint sale" is a(n) a. illegal practice of selling mutual fund shares to investors without disclosing all of the risks involved when investing b. Prohibited practice that registered reps should refrain from as the result is usually an inferior transaction for the customer c. Industry recognized sales practice that reward the efforts of mutual fund salespeople d. a sales practice that registered reps should practice as the result is usually a better deal for the customer

b. Prohibited practice that registered reps should refrain from as the result is usually an inferior transaction for the customer a "breakpoint sale" is a prohibited sales practice which often results in a customer receiving a disadvantageous price when purchasing mutual fund shares


Ensembles d'études connexes

Measuring Health and Development

View Set

Chapter 23 Plant Evolution and Diversity - BSC 111

View Set

Chapter 4, Lesson 1, pages 136-139

View Set

Live Virtual Machine Lab 10.1: Module 10 General Network Attacks

View Set

Chapter 1 Quiz Sports Psychology

View Set

Prepare to Visit: Miramar Community Garden Visit

View Set